You are on page 1of 77

2

WIN IN TS!

Editor-in-Chief Ann Guo


Co-Editors Ree Zhang, Emily Wang, Bianca Chan &
Sherry Zhang

__________________________________________________________________________________
Copyright 2022 Into College Pty Ltd All Rights Reserved.
3

CONTENTS
1. Introduction
a. What is TS
b. How to approach TS
c. How to use this book
2. Practice Paper
3. Topics
a. Logical Reasoning
i. Interpreting Information
ii. Euler/Syllogisms
iii. Logic Lists
iv. Logic Tables
b. Arguments
i. Strengthening Arguments
ii. Weakening Arguments
iii. Mistakes, Flaws, Fallacies
c. Drawing Conclusions
d. Quantitative reasoning
i. Numerical Working
ii. Graphs and Diagrams
e. Abstract reasoning
i. 2D Spatial Awareness
ii. 3D Spatial Awareness
iii. Sequence Recognition
4. Practice question answers

__________________________________________________________________________________
Copyright 2022 Into College Pty Ltd All Rights Reserved.
4

EDITOR’S NOTE
This book is for students to have a better understanding of what Thinking Skills is and how to
answer the different types of questions they have face in a test. Even though Thinking Skills is
a challenging subject, we believe that through consistent practice and developing critical
thinking, students will be able to gain skills far beyond just the contents of the test itself.
Thinking Skills is a very new subject when compared to the other tests. Despite it replacing
General Ability, there are still substantial differences between these two tests. Thus, it is crucial
to practice specific question types, revise past papers and sample papers and gain a strong
understanding of the types of logic used to solve the questions.
Best wishes to all students undertaking Thinking Skills in their exams!

THE EDITORS
Ann is the Director of Thinking Skills at Into College, with experience teaching Thinking Skills,
General Ability and English. She studies science, with a focus on Neuroscience and
Biochemistry. She graduated from Abbotsleigh School for Girls, being an Honour Roll student
throughout her years of study (2015-2019). She was invited to the Earth and Environmental
Science Olympiad (2019) being one of the Top 24 students in the nation and an alumni of the
National Youth Science Forum (2019). She is a Denison Summer Science Scholar (2022) at
the University of Sydney.

Ruiyi (Ree) is a Thinking Skills tutor and resource developer at Into College focused on
primary tutoring of Maths, Thinking Skills and General Ability. In her 4 years of tutoring, she
was able to help many students achieve excellent and ideal results in the OC Placement Tests,
Scholarship Tests and Selective School Exam. She graduated from Hornsby Girls’ High School,
one of Sydney’s top performing girls’ schools, with their cohort ranking 5th in the HSC exams.
She currently studies a Commerce and Advanced Science double degree at UNSW with a focus
on Business Economics and Biotechnology with plans to pursue Biotechnology related honours.

Emily is a Thinking Skills Tutor at Into College, with 3 years of tutoring experience in Maths,
English and Thinking Skills. In the past, she has helped many students achieve outstanding
performance in the OC Placement Test, Selective School Exam and HSC exams. She graduated
from Hornsby Girls’ High School, one of Sydney’s top performing girls’ schools, with their
cohort ranking 5th in the HSC exams. Currently, she studies Commerce and Advanced Science
at UNSW, with a focus on Marketing and Psychology.

__________________________________________________________________________________
Copyright 2022 Into College Pty Ltd All Rights Reserved.
5

Bianca is a Thinking Skills Tutor at Into College, with experience in developing resources and
teaching Thinking Skills, as well as 2 years of tutoring experience in Maths and Biology. She
graduated with a Bachelor of Biomedicine at the University of Melbourne with First Class
Honours and progressed to an Honours Project on metastatic colorectal cancer. She was Dux
of VCE Biology at John Monash Science in 2018 and recipient of the J.E. Taylor Honours
Award for clinical pathology.

Sherry is a Thinking Skills Tutor at Into College with 4 years of teaching experience in primary
and high school, as well as professional development courses for university students. She
studies Aerospace Engineering and Commerce (Marketing) at UNSW. She graduated from
Sydney Girls High School and has been awarded the Sydney Girls Medallion, received after 6
consecutive years of outstanding academic achievement, leadership and community service. In
university, she has been a finalist in the UNSW Founders New Wave program and won the
Mastercard & BITSA Innovation Challenge.

__________________________________________________________________________________
Copyright 2022 Into College Pty Ltd All Rights Reserved.
6

INTRODUCTION
WHAT IS THINKING SKILLS
Thinking Skills is a collection of questions involving real-world issues that challenge students’
abilities in critical thinking and solving novel problems.
Following the style of the Thinking Skills Assessment from Cambridge Assessment
Admissions Testing, Thinking Skills not only develops valuable techniques such as logical
reasoning, decision making and analysing arguments, but also fosters students’ broader
abilities in critical thinking, interpreting information and reasoning from first principles.
There are 2 main categories of questions in TS: critical thinking and problem solving. The main
topics that this book covers can also be separated based on these categories.
Overall, Thinking Skills involves a range of skills and question types that challenge students
to think critically about how to interpret and effectively reason with new information. However,
given the right practice, these foundational abilities will benefit students throughout high
school and beyond, even appearing in subsequent exams such as the UCAT.

Critical Thinking Problem Solving

This category of question can be thought of This category of question is about numerical
as verbal reasoning and includes topics reasoning, looking at tables, diagrams,
involving logical explanations, arguments graphs and solving problems with numbers
and finding conclusions from a paragraph. or shapes.

This evaluates students ability to read from a This involves being able to find similarities,
given source (reading comprehension) and follow procedures and use relevant
use the underlying logic given to come to a mathematical thinking to come to numerical
solution. or spatial solutions.

1. Logical Reasoning 4. Quantitative reasoning


a. Interpreting Information a. Numerical working
b. Euler/syllogisms b. Graphs and diagrams
c. Logic lists 5. Abstract reasoning
d. Logic tables a. 2D Spatial Awareness
2. Arguments b. 3D Spatial Awareness
a. Strengthening Arguments c. Sequence Recognition
b. Weakening Arguments
c. Mistakes, Flaws, Fallacies
3. Drawing Conclusions

__________________________________________________________________________________
Copyright 2022 Into College Pty Ltd All Rights Reserved.
7

HOW TO APPROACH THINKING SKILLS


Thinking Skills questions use a range of scenarios to present the underlying logic. As TS deals
with novel problems, this will only help with the patterns we use to solve them and not in
memorizing or rote learning the questions themselves. Thus, while having some general
knowledge about different relevant world issues can be useful, the key to approaching Thinking
Skills efficiently is to focus on building up students’ higher order skills of critical thinking,
interpolation, extrapolation and interpreting information.

Generally, solving questions will not require any in-depth background knowledge. However,
good critical thinking and evaluation is needed to arrive at the right answer. Thus, being able
to effectively digest information is key. These are skills that can be used in every single
question and are very important.
This involves adapting to new information and processing it correctly:
• Reading the question carefully and clearly
o Using each part of the question
• Finding the main argument/points of important information
o Find relevant information to help us
• Being able to evaluate accurately
o Also read the answers carefully
Being able to do this is what really makes a student good at answering Thinking Skills questions.

HOW TO USE THIS BOOK


This book covers:
• An introduction to Thinking Skills
• Practice paper
• Question topics and explanations
• Practice problems
Each topic has been chosen based on the current available papers and question types seen in
Thinking Skills. Each topic corresponds to a given question type, covering:
• Meaning & definition
o Background knowledge and information on this topic
o Importance and relevance of this topic
• Skills and tips
o What skills does this topic and its question type involve?
o What tips are there to solving this type of question?
• 1 worked example with a step-by-step guide to solving
• References to currently available official Thinking Skills paper
• 3 practice questions for students to work on
__________________________________________________________________________________
Copyright 2022 Into College Pty Ltd All Rights Reserved.
8

PRACTICE PAPER
1 5 kids, 2 girls and 3 boys were participating in a 400m race at their school’s sports
carnival. Unfortunately, Jenny tripped and had to stop hte race halfway and go to the
school nurse to get her ankle checked. While running the race, Adam saw 2 people,
one boy and one girl pass the finish line before him. When the race was over, the
school announced that a boy had come first in the 400m race. Based on the given
information, what must be untrue?

A. Adam had finished the race third


B. Jenny did not finish the race due to external circumstances
C. A girl finished the race after Adam
D. Jenny should get a participation prize for her efforts

2 At an auction there are four cars for sale. The Mercedes costs more than the Holden,
the Bentley costs more than the Mercedes, and the Holden costs more than the BMW.
Which is the most expensive car in this auction?

A. Bentley
B. Mercedes
C. Holden
D. BMW

3 Malthus: Increasing the food supply can only alleviate human suffering temporarily. If
we ensure plenty of food is available for everything, then people will have more
children, and the human population will grow so large that there will be another
famine, and the suffering will continue.

Which of the following, if true, most strengthens Malthus’ argument?

A. Plentiful food for everyone is the primary enabler for people to live long lives
and have children
B. Humans can decide to have children based on factors other than whether they
can feed the children
C. Famine and suffering are essentially one and the same
D. Famines have always been cyclical in nature

__________________________________________________________________________________
Copyright 2022 Into College Pty Ltd All Rights Reserved.
9

4 Most railway stations have digital clocks that display the time in 24-hour format. I
recently undertook a train journey from Exeter to Aberdeen. As the train left Exeter,
the clock on the platform showed:

I lost track of time as I read a book and then dozed. I eventually became aware of the
train coming to rest and the guard announcing 'Carlisle’.

I opened my eyes, and found that I could only see the top of the platform clock
(because of the hoarding in the way) as follows:

Approximately what time was it, to the nearest hour, when the train stopped at
Carlisle?

A. 1 o’clock
B. 2 o’clock
C. 3 o’clock
D. 4 o’clock

5 Ann owns a gift shop where she makes and sells the gifts herself. Her shop is open
from Monday to Friday and for half a day on Saturday. She makes the gifts in store
and can make more or less per day depending on the number of customers. Last week,
she had 32 customers on Monday, Tuesday and Thursday, 53 customers on Friday and
12 customers on Wednesday. On Saturday, she managed to see over 100 customers
even though she was only open for half a day. Below is a table showing the amount of
gifts she makes depending on the number of customers:

Numbers of customers Number of gifts


Less than 20 80
Less than 50 60
Less than 70 35
More than 70 5

How many gifts did she make this week?

A. 260 gifts
B. 300 gifts
C. 295 gifts
D. 320 gifts

__________________________________________________________________________________
Copyright 2022 Into College Pty Ltd All Rights Reserved.
10

6 Frazer's uncle says this to him all the time: For you to get a good job when you are
older, you need good grades at school now for entering university in order to receive
your degree.

Frazer: Getting good grades now in school means I am on track to get a good job later
in life.

Frazer's brother: If I can get a degree without getting good grades, then I can still get a
good job!

If the information above is true, then whose reasoning is correct?

A. Frazer only
B. Frazer’s brother only
C. Both Frazer and his brother
D. Neither Frazer nor his brother

7 Jennifer is making tarts for her daughter’s birthday. She wants to make a total of 25
tarts and the recipe she is following allows her to make 10 tarts each time. For every
10 tarts, she needs 200g flour, 100g sugar, 150g butter and 1 egg. At home, she has
800g flour, 300g sugar, a dozen eggs and 300g of butter. She realised she just short on
some ingredients to make her 25 tarts. What is she missing and how much is she
missing?

A. Butter, 150g
B. Butter, 75g
C. Sugar, 100g
D. Sugar, 50g

8 Everyone in the chess club registered for the school chess competition. Kamila is part
of the chess club and registered, because she is looking forward to only playing other
chess club members in the school chess competition.

Which of the statements best addresses the mistake in Kamila’s decision to register?

A. Kamila registered because everyone else in the chess club also registered
B. Most of the students in the school are not part of the chess club
C. Not everyone in the school chess competition is part of the chess club
D. The school chess competition is organized by the chess club

__________________________________________________________________________________
Copyright 2022 Into College Pty Ltd All Rights Reserved.
11

9 An express train is departing from Central at 1:30pm and arriving at Newcastle at 3:30
pm. An all stop train departed at the same time as the express train from Central and is
also heading towards Newcastle. However, during the journey, all stop train broke
down for 30 minutes. The express train arrived five times as fast as the all stops train
and if each train travels at constant speed, when did the all stops train arrive at
Newcastle?

A. 10:30pm
B. 11:30pm
C. 12:00am
D. 12:00pm

10 To hike a long continental trail walk, you must be in excellent physical


condition and have perseverance to overcome hardships.

Maria: “Luke has a lot of perseverance – he has hiked many long trail walks before –
but he is no longer in good physical condition as he has not exercised in the past year.
Hiking is no longer a good option for him.”

Nolan: “Jasmine is very fit and loves exercise. She also makes sure to double-check all
her maths problems and works to solve them even when they are challenging. She’s
sure to be great at hiking continental trail walks.”

If the first statement above is true, whose reasoning is not correct?

A. Maria only
B. Nolan only
C. Both Maria and Nolan
D. Neither Maria nor Nolan

11 To fully understand a great work of art, you must first understand the artist’s
background and then see how it has been reflected in the artwork.

Melinda: “Understanding the artist's background is more important than appreciating


the artwork.”

Arthur: “Appreciating the artwork is more important than understanding the artist's
background.”

If the information in the box is true, whose reasoning is correct?

A. Melinda only
B. Arthur only
C. Both Melinda and Arthur
D. Neither Melinda nor Arthur

__________________________________________________________________________________
Copyright 2022 Into College Pty Ltd All Rights Reserved.
12

12 A cycling instructor said that it is important to wear helmets when cycling, and that
children who wear helmets are less likely to suffer severe head injuries if an accident
occurs.

Which one of these statements, if true, best supports the instructor’s claim?

A. Bicycles are slower than cars


B. Good helmets are more expensive than before, however their quality has
improved greatly
C. Reports of accidents find that helmets have prevented head injuries when
compared to similar incidents without helmets
D. Cycling is a dangerous sport

13 Mahogany wood can be used to make furniture and musical instruments, like
guitars and pianos. Jack’s chair and guitar are made out of the same wood.

Jack: “If the only wood I buy for my house is mahogany, then my chair must be
mahogany.”

Jill: “If what Jack says is true, then his guitar must also be mahogany.”

If the statement above is true, whose reasoning is correct?

A. Jack only
B. Jill only
C. Both Jack and Jill
D. Neither Jack nor Jill

14 Sammy is moving schools so she decided to buy everyone in her class a goodbye
present. There are 30 kids in her class, including her as well as 2 teachers. For the kids,
she decided to buy everyone a Christmas card and a small candy cane each. For her 2
teachers, she wanted to buy them a box of chocolate each. She also wanted to have
both boxes of chocolates wrapped with gift wrapping. Below is a table showing the
prices of gifts:

Gift Price
Cards (pack of 30) $12
Candy cane (pack of 15) $10
Chocolates $15
Gift wrapping (per present) $5

How much did she spend on the gifts in total?


A. $62
B. $72
C. $57
D. $67

__________________________________________________________________________________
Copyright 2022 Into College Pty Ltd All Rights Reserved.
13

15 Thomas is flying to Rome for the weekend.His flight is scheduled to arrive Sunday at
07:23 and to leave on Monday at 22:50. When in Rome, he will use a shuttle bus each
way between the airport and the city centre. The shuttle bus timetable is as follows:

Airport to Centre of Rome


Depart 08:00 08:20 08:40 09:00 09:20 09:40 10:00
Arrive 08:25 08:45 09:05 09:25 09:45 10:05 10:25

Centre of Rome to Airport


Depart 20:10 20:35 20:50 21:10 21:30 21:50 22:10
Arrive 20:35 20:55 21:15 21:25 21:55 22:15 22:35

His arrival in Rome has been delayed by 1 hour 45 minutes. It will take him 30
minutes to collect his bags and clear customs on arrival. The airline requires him to be
at the airport at least 2 hours before his return flight is scheduled to leave.
What are the times of the buses he will take to and from the city centre?

A. 08:40 and 21:10


B. 9:40 and 20:10
C. 9:40 and 20:30
D. 10:00 and 20:10

16 Jess needs various lengths of pipe to complete some home improvement work.
Specifically she needs one pipe each of lengths 2 m, 4 m, 6 m, 8 m, and 10 m. She can
buy pipe in lengths of 3m, 5m, 7m and 9m and cut them to the sizes she needs. Pipe
costs $1 per metre whatever the length of pipe bought. What is the total cost of the
pipe she needs to buy?

A. $26
B. $27
C. $30
D. $33

17 Five children did a test with five difficult questions to answer. Haily was faster than
Gabriella, but got three questions wrong. Adam got all the questions right, but took the
longest. Natalie finished second, but got two questions wrong. Gabriella got more
questions right than Natalie, but finished after her. Kamilla wasn’t as fast as Natalie,
but Natalie was not as fast as Hailey.

If all the above statements are true, which of the sentences below cannot be true?

A. Gabriella got fewer questions right than Adam


B. Hailey was faster than Kamilla
C. Hailey was not first to finish
D. Hailey got fewest questions right

__________________________________________________________________________________
Copyright 2022 Into College Pty Ltd All Rights Reserved.
14

18 Three graduate school friends, Anna, Bec and Carol, graduated successfully. Being in
the same program, the three often worked as a team on group assignments. Anna
earned the special award of “distinction” when she graduated. Carol and Bec, although
receiving their degrees, did not earn the same special award A fourth student in the
same graduate program, Debby, often said that the graduate program was poorly
designed and not difficult at all. Debby did not graduate - instead, she was advised by
her teacher to leave the program because her work was below acceptable standards.

Given this information, which of the following must be true?

A. Carol and Bec deserved a “distinction” award like Anna


B. Bec was jealous of Anna’s success in getting a “distinction”
C. Debby’s work quality was the lowest out of everyone
D. Because Anna received a “distinction” she will work on more advanced
projects

19 Marcy and Fei were eating out at a fast-food restaurant when they noticed a sign.

Marcy: “Then all food that has been returned today is either contaminated or there are
missing ingredients but not both.”

Fei: “Surely all contaminated food gets returned to the restaurant.”

If the information in the box is true, then whose reasoning is correct?

A. Marcy only
B. Fei only
C. Both Marcy and Fei
D. Neither Marcy nor Fei

20 A dishonest act, such as stealing money or cheating in school, may happen for many
different reasons. But the reasons may reflect the personality of the person involved.
One may steal in order to show off. Another, to get money to support a hobby. One
may cheat in order to avoid punishment for academic failure; another, because of
personal ambition.

Which of these statements, if true, most strengthens the argument?

A. There are many types of dishonest acts that can be performed


B. People may have different motivations, but dishonest acts still should not be
performed
C. It is fine to perform dishonest acts, even if there is no good reason to
D. Addressing each dishonest act individually works better than general
punishments

__________________________________________________________________________________
Copyright 2022 Into College Pty Ltd All Rights Reserved.
15

21 Bacta is being freely dispensed. Scorch says there’s nothing wrong with getting some
extra Bacta. It’s well known that Bacta is good for your health, and you can always
save it to use later. Besides, if it is being dispensed freely, then surely there’s plenty of
it to go around anyway.

Which statement, if true, most weakens Scorch’s argument?

A. Bacta tends to degrade quickly, and once it has degraded, it is not as effective
B. Bacta is only freely dispensed in places where it is desperately needed
C. Most people’s health does not actually benefit from Bacta
D. Disposing of excess Bacta is quite expensive

22 6 kids, Amy, Becky, Carl, Dan, Eli and Fred all decided to enter their school’s spelling
competition because their favourite subject was English. There are 2 rounds in total
and the final overall winner gets a prize. In the first round all 6 kids compete but only
the top 3 can move on to the next. Eli and Amy unfortunately did not make it into the
second round. It was observed that 1 girl and 2 boys made it into the second round, and
the final winner was a girl. Fred and Dan got participation awards for making it into
the second round.

Based on this information, what must be true?

A. Fred and Dan deserved to win the grand prize


B. Whoever wins the grand prize means that they hate maths
C. Amy definitely came last
D. Carl didn’t make it into the second round and may have come 4th

__________________________________________________________________________________
Copyright 2022 Into College Pty Ltd All Rights Reserved.
16

23 Various shapes may be made by overlapping two transparent squares, for


example the dark shaded hexagon shown below:

Which one of the following shapes cannot be made as the overlap of two transparent
squares?

24 Some people explain recurring dreams (those that you keep on having time after time)
as messages from your unconscious. Some messages are too important to send just
once in case you don't understand them the first time or don't realise their importance.
So, a recurring dream must contain a message that is worth repeating over and over.

Which one of the following identifies the flaw in the reasoning in the above argument?

A. Even if an explanation makes sense it is not necessarily true


B. Not everyone remembers the dreams they have had
C. Sometimes we may dream that we have had a dream before
D. Not all dreams contain important messages

25 A chiropterist (scientist who studies bats) wrote a book based on his past research and
observations that states some mammals are bats. It is also known that all bats can fly
just like birds. What valid conclusion can a student make based on the chiropterist’s
book?

A. Some bats are birds


B. Some mammals can fly
C. All bats are birds
D. No valid conclusion can be drawn

__________________________________________________________________________________
Copyright 2022 Into College Pty Ltd All Rights Reserved.
17

26 Below is Tina Linette Fu’s brass paperweight, which shows his initials.

Which one of the following is not a side view of Tina’s paperweight when it is placed
flat on a table (either side up)?

27 If Hilda is late to the bus stop, then she will miss the bus and be late to school. If she is
late to school, then she will miss roll call and need to get a late slip from the office. If
she does not miss roll call, then she might get to play with friends. Otherwise, she
won’t be able to do so.

If the above statements are correct, which of the following is not possible?

A. Hilda was late to the bus stop, but played with friends
B. Hilda did not miss the bus, but played with friends
C. Hilda did not miss roll call and did not get a late slip
D. Hilda was late to school, but still missed roll call

28 In a class, there are 30 students who like chocolate. 26 students like vanilla. 20
students like neither. If there are 70 people in the class, how many students like
chocolate and vanilla?

A. 4
B. 6
C. 10
D. 20

__________________________________________________________________________________
Copyright 2022 Into College Pty Ltd All Rights Reserved.
18

29 Some mammals live in water while other mammals live on land. An aquatic mammal
is a mammal that lives partly or entirely in water (freshwater or saltwater), while a
marine mammal is a mammal that lives in the ocean and in saltwater (not freshwater)
habitats.

Marcy: “If you see a mammal in the ocean, it must be a marine mammal.”

Patty: “And if you see an animal in the ocean, and you know it isn’t a marine mammal,
then it must not be an aquatic mammal.”

If the first statement above is true, whose reasoning is correct?

A. Marcy only
B. Patty only
C. Both Marcy and Patty
D. Neither Marcy nor Patty

30 The idea of cleaning your study space in real life should also be applied to your
computer and digital environment, such as sorting important files and deleting
unnecessary clutter. This leads to higher productivity and more efficient working in the
same way decluttering your desk and study space does. Which best captures the main
conclusion?

A. Higher productivity and more efficient working are beneficial


B. Organising a digital workspace is just as important as physical organisation
C. The only sorting files and deleting unimportant files are digital decluttering
D. Decluttering is necessary to improve work quality

31 In a survey of television viewers, everyone who liked volleyball liked skiing. Everyone
who liked skiing liked cycling, but no one who liked skiing liked gymnastics. David,
Alex, Lesley and Jennifer all took part in the survey. Based on the above information,
which of the following must be true?

A. If David likes cycling, he also likes skiing.


B. If Amanda does not like gymnastics, she does not like cycling.
C. If Lee does not like volleyball, he does not like skiing.
D. If Jess likes volleyball, she does not like gymnastics.

32 Four students line up for their sports team photo at photo day. They must find out the
order of heights so the tallest can stand in the middle to spots, with the shortest
students on the outside to spots. Anna knows she is shorter than Celeste. Bertrand is
taller than Desmond. Anna and Desmond are similar heights. What is the order of their
line up for the photo, from left to right?

A. Anna Bertrand Celeste Desmond


B. Bertrand Anna Desmond Celeste
C. Celeste Desmond Anna Bertrand
D. Desmond Anna Bertrand Celeste

__________________________________________________________________________________
Copyright 2022 Into College Pty Ltd All Rights Reserved.
19

33 Birds of the tit family are very fond of tearing paper - wallpaper, newspaper, notices
and bank notes included. They make no use of the paper they have torn. We might
explain this puzzling behaviour by noting that the technique that they use for tearing
paper is the same as their method for stripping tree bark to search for insects to eat.
This indicates that tits that tear paper are searching for food.

Which of the following, if true, would most weaken the above argument?

A. On many occasions, tits reject food on bird tables to tear paper


B. Tits tear paper when there is no food
C. 80% of householders who reported paper tearing behaviour did not feed the
birds
D. Tits use moss from trees and wool from dead sheep for nesting materials

34 Four friends are in the same maths class. For their maths project, they must find out the
order of their heights. Unfortunately, Jess is sick and did not come to school the day
they started the project. Charlotte knows she is shorter than Jess. Matt is taller than
Tim. If Tim says that Jess is shorter than him, then who is the tallest?

A. Charlotte
B. Matt
C. Tim
D. Jess

35 My TV remote control is shown below. All the buttons stick out from the body of the
remote control.

When viewed from the direction of the arrow, which one of the diagrams below is not
a possible view of the remote control?

__________________________________________________________________________________
Copyright 2022 Into College Pty Ltd All Rights Reserved.
20

36 A pentomino is a tile made up of 5 squares which are joined along their edges. Four
pentominoes have been placed together in the pattern shown below.

Two more pentominoes are to be placed to fill the gap. One of them is a new shape,
while the other is a repeat of one of the pentominoes already positioned. The new
shape is shown below.

Which is the other pentomino that is to be placed in the gap?

37 The town council did a survey for 1000 people to find out what new facility the
residents would most like to have built in the town. The results are shown in the table:

Facility Votes
Leisure centre 513
Library 231
Museum 120
Swimming pool 118
Other suggestions 18

The results were also shown on a pie chart, but the regions were not labelled. Which
one of the following correctly represents the result?

__________________________________________________________________________________
Copyright 2022 Into College Pty Ltd All Rights Reserved.
21

38 Three friends Stella, Daisy and Jim entered a cycling race in their local community.
During this race, they each had a different small accident (falling off, stuck wheel and
punctured wheel) that caused them to lose some time. These accidents happened at
three different points of the race; the beginning, middle and end.

It is known that neither of the girls fell off their bike, and Daisy finished in the best
position out of the friends. The rider who had their accident at the end of the race
finished second from last. Stella’s mishap was the stuck wheel during the middle of the
race. Who finished last in the race?

A. Stella
B. Daisy
C. Jim
D. Not enough information

39 Anand: The school newsletter often publishes short stories written by honour roll
students. The editor of the school newsletter says that the honour roll achievement of
the student is one of the most important factors in selecting which stories to go in the
newsletter. Since Silson, one of my friends, has one of his stories selected for the
school newsletter, he must be an honour roll student.

Which of the following describes the mistake that Anand has made?

A. He doesn’t consider the fact that honour roll students can have different
achievements
B. He treats one main factor of story selection as a condition that must be met
C. He does not use an accurate source for evidence
D. He believes that being on the honour roll and having a story selected are results
that likely stem from the same characteristics of students

40 Lauren lost her red ink marker in art class when they were drawing, earlier today.
Lauren: “I’m trying to find my red marker because I lost it in art class today. Have you
seen it?”
Daniel: “I saw a red marker with a transparent cap in the art class on the desk, it must
be yours!”

Which one of the following sentences shows the mistake Daniel has made?

A. There might be more than one red marker in the art class
B. Lauren might not like the colour red
C. Lauren might have misplaced her marker, instead of losing it
D. Even if the marker is a red marker, it might not have a transparent cap

Please contact into College via WeChat for the solutions to this practice paper.

__________________________________________________________________________________
Copyright 2022 Into College Pty Ltd All Rights Reserved.
22

LOGICAL REASONING

__________________________________________________________________________________
Copyright 2022 Into College Pty Ltd All Rights Reserved.
23

INTERPRETING INFORMATION
Interpreting information is one of the foundational topics of Thinking Skills, relevant in a
variety of uses from reading passages to quantitative reasoning questions and in our everyday
life. To be able to solve these kinds of problems, students must be able to think rationally to
understand the meaning of the given information.
There are three main types of information students can interpret: visual, oral and written. Visual
information includes content such as graphics and images, while oral information requires
auditory focus such as listening to speeches or a lecture. Finally, written information is the
most common in exams as it is information that is written on the sheet to be read and understood
by the student. Often, these information types can be combined, such as in a video where there
is both visual and oral information, or a comic where there is both visual and written
information.
Interpreting information helps students to understand the characteristics of the question,
important details in the question and the relationships between the information given.pic allows
students to better note take and understand concepts better, and teaches strategies to use when
students have trouble understanding the content.

SKILLS & TIPS


When interpreting information, some key skills include:
• Looking out for keywords and subject. Some keywords include but, however, some,
most, all, never etc.
• Locate the topic sentence and determine the key point
• Filter out filler sentences (i.e. sentences that are not important to the overall message)
Summarising information is also related to interpreting information questions, and also helps
in finding important information within long texts. Two techniques for summarising are:
• Skimming, which is a method of picking out key words throughout the passage to
gain a general understanding of the content
• Scanning, which is a method of looking for key words or phrases to find out specific
information

__________________________________________________________________________________
Copyright 2022 Into College Pty Ltd All Rights Reserved.
24

WORKED EXAMPLE
After the school term ended, Amy’s family decided to go on a holiday and have some fun. Amy
left Sydney and went to Melbourne four days ago and her Mum told her to pack summer clothes
and a hat because it was very hot the day she left. They decided to return on Sunday which is
four days from now so she has enough time to do her summer holidays homework.

Based on the above information, which of the following statements must be true?

A. Today is Thursday.
B. It was very hot last Saturday
C. Amy left for her destination last Monday
D. It was very hot yesterday
Firstly, we want to find the main information the question tells us. Amy left four days ago, and
will return on Sunday, which is four days from now.
As Sunday is four days from now, we can reason that today must be Wednesday. That means
if she left four days ago from today (Wednesday), then she must have left last Saturday.
Therefore, A and C are incorrect as the days do not match with the information given. D cannot
be certain as there is no information regarding that information. Thus, only Option B is correct,
as the question tells us it was hot when she left, which was last Saturday.
Note that we also filter out irrelevant information, such as the fact that she was leaving from
Sydney to Melbourne, or that she had to come back to do her homework.

QUESTIONS
All Hibs are Hobs. Some Habs are Hibs. Therefore, all Habs are Hobs. Is this true or false?
A. True
B. False
C. Uncertain
D.

John is visiting a friend on the weekend. He wishes to bring gifts for the family and he is buying
all of them from the same store. He is buying a box of chocolate for the family to share and a
game for each of the three children. He also wants to have the chocolate wrapped at the store.
How many presents did he buy?
A. 2
B. 4
C. 5
D. 1
__________________________________________________________________________________
Copyright 2022 Into College Pty Ltd All Rights Reserved.
25

Jenny is moving schools next year so she decided to buy everyone in her class a small
Christmas present as a goodbye gift. There are 30 kids in her class, including her and 2 teachers.
For the other kids in her class, she decided to buy everyone a Christmas card and a small candy
cane each and for her 2 teachers, she wanted to buy them a box of chocolate for each of them
too. She also wanted to have both boxes of chocolates wrapped with gift wrapping. Below is a
table showing the prices of gifts:
Gifts Price
Cards (pack of 30) $12
Candy Cane (pack of 15) $10
Chocolates $15
Gift wrapping (per present) $5

How much money does she spend in total for everyone’s presents?
A. $72
B. $62
C. $57
D. $67

__________________________________________________________________________________
Copyright 2022 Into College Pty Ltd All Rights Reserved.
26

EULER/SYLLOGISMS
Syllogisms are a form of reasoning in Thinking Skills in which a conclusion is drawn from two
given or assumed premises. Syllogisms are based on deductive reasoning, which is the logical
process where the conclusion is based on these premises that are generally true.
This logical construct has two criteria and one conclusion, usually in this form:
1. Major premise
2. Minor premise
3. Conclusion
Deductive reasoning questions are both logically true and realistically true, following an idea
and observation to form a conclusion. This is opposed to inductive reasoning, where
observations come first, followed by analysis which lead to a theory, and thus is logically true
but not always realistically true.
Euler diagrams can be used to determine the validity of arguments when premises contain key
words like all, some and no:
• All A are B
• No A are B
• Some A are B
• Some A are not B
With these types of questions, the argument is valid only if all variations of the Euler diagram
illustrate the conclusion made. If any of the Euler diagrams contradict the conclusion, then the
argument is invalid.

SKILLS & TIPS


Solving syllogisms and Euler questions include interpreting information and deductive
reasoning skills. It is also important to identify key words to help draw conclusions, such as all,
some or no.
When drawing Euler diagrams, it is important to first identify the number of categories in the
question. Then draw circles for each premise one by one, noting how the premises relate with
each other. When it is complete, don’t forget to check if all premises have been included, and
that the number of circles equals the number of categories in the question.

__________________________________________________________________________________
Copyright 2022 Into College Pty Ltd All Rights Reserved.
27

WORKED EXAMPLE
All primates are curious. Some primates are carnivores. Therefore, some carnivores are curious.
If these statements are true, is the conclusion “some carnivores and curious” true or false?
A. True
B. False
This question can be solved by drawing a Euler diagram, broken down into the following steps:
1. Number of categories: 3 (Primates, carnivores, curious)
2. Draw the diagram for the first premise: All primates are curious. Thus, the primates
circle will exist within the curious circle.

3. Draw the diagram for the next premise: Some primates are carnivores. Thus, the
carnivores circle will be partially in the primates circle. This can be drawn in two ways:

4. From the Euler diagrams drawn, we can see that in both cases that “some” primates
overlap with carnivores, as required in the premise. Since it can be seen that
carnivores also overlap with curious in both cases, the conclusion must be true.

__________________________________________________________________________________
Copyright 2022 Into College Pty Ltd All Rights Reserved.
28

QUESTIONS
According to the following diagram, which of these statements is invalid?

A. Those who like hamsters also like rabbits


B. Those who like hamsters also like cats
C. Some who like cats also like dogs
D. Some who like dogs also like hamsters

An experiment was conducted in science class about plants. It is known that all plants including
fungi need water to grow. Through the experiment, it is also observed that fungi do not need
sunlight to grow while all plants do. What is a valid conclusion that can be drawn?
A. Plants need either water or sunlight, it does not need both
B. Fungi are plants
C. Fungi are not plants
D. Both fungi and plants need sunlight to grow

Arthur’s school is doing a survey with all the students to determine which new classes to
introduce. Everyone who wanted to do woodwork wanted to do pottery. Some people who
wanted to do pottery wanted to also do textiles. No one who picked woodwork wanted to do
textiles. If the above information is correct, which of the following statements are correct?
A. If I picked textiles, I must have also picked gardening.
B. I picked pottery, so I must have picked textiles
C. If I did not pick textiles, I might have picked woodwork
D. The above information is all correct

__________________________________________________________________________________
Copyright 2022 Into College Pty Ltd All Rights Reserved.
29

LOGIC LISTS
Logic lists questions are questions where a list of information is given, and a ranking or order
must be determined to answer the question. Solving these types of questions is crucial in
Thinking Skills as it is prevalent in many questions and builds on a range of foundational skills
such as interpreting information to draw conclusions.
Interpreting information is crucial for this question as it is necessary to understand the
characteristics of the question, important details in the question and the relationships between
the information given. Often, deductive reasoning is needed to draw conclusions between
information that is not explicitly given. Finally, visualising information is required to place
certain elements above or below another to help solve the problem.
Many questions involve making lists, such as those involving height, speed, and test results. It
is important for students to be familiar with logic list questions, as these questions can be easily
solved in exams as a definite answer can usually be deduced with high certainty.

SKILLS & TIPS


When solving logic list questions, it is important to read the question carefully and use all the
relevant information given. This is an extension of interpreting information, as it requires
students to identify key words and filter out filler sentences.
When solving logic list questions, it is often helpful to draw a list, line or a row of boxes to
visualise where each element should be placed. When there is extra information, it is also useful
to note down the relationship between each rank; for example, for a height question, the
difference between each rank can be recorded as well.

WORKED EXAMPLE
Five friends are doing homework together.
Katie knows she handed in her work before two people, but got the lowest mark. John was the
fastest and got all the questions right. Jennifer took the longest and got all by 1 question right.
Chris finished after Katie but did not tell anyone his mark. Terry got 2 wrong in total.
If the above information is true, which of the following statements is untrue?
A. Jennifer had a higher mark than Katie
B. Terry handed in his homework after Katie
C. Chris was 4th to hand in his homework
D. John had a higher mark than Jennifer
This question is an example of a logic list question with two types of information. One is the
speed that the students did their homework, and the other is the marks that they received for
the homework.

__________________________________________________________________________________
Copyright 2022 Into College Pty Ltd All Rights Reserved.
30

By analysing the information given and options given in the multiple choice, we can work with
ranking their speeds primarily as that information is more prevalent.
The rankings can be deduced as follows:
• Katie is 3rd in speed as she handed in her work before 2 people, and 5th for mark as
she received the lowest mark.
• John was 1st for speed and mark as he was the fastest and got all the questions right.
• Jennifer was last (5th) as she took the longest, and 2nd for marks as she only got 1
question wrong.
• Chris finished after Katie - this means he is 4th, as that is the only position left after
Katie (we now know that Jennifer is 5th)
• This means that Terry was 2nd, as that is the only position left.
Then, we can analyse the options. A, C and D are correct as deduced.
Option B is untrue as it can be solved from the information that Terry handed his homework in
2nd, and Katie handed her homework in last.

QUESTIONS
John and his 3 brothers James, Jack and Joe are playing rock, paper, scissors. They agreed that
the person who loses the most rounds will wash the dishes. John knows that he won the most
rounds overall and that James won 3 less rounds than him. Jack knows that he won 1 more
round than James. Joe won 4 rounds less than John.
If the information given is true, who will be washing the dishes?
A. John
B. James
C. Jack
D. Joe

5 students are taking a quiz for History. Andrew knows he handed in his work before 2 people
but got the lowest mark. Bob was the fastest and got all the questions right. Samantha took the
longest and got all but 1 question right. Marlene finished after Andrew but did not tell anyone
her mark. Tom got 2 wrong in total.
If the above information is true, which one of the following statements is true?
A. Marlene placed 2nd in terms of marks
B. Tom was the 2nd student to hand in his quiz
C. Samantha came last in marks and in handing in the quiz
D. Marlene was the 3rd student to hand in the quiz

__________________________________________________________________________________
Copyright 2022 Into College Pty Ltd All Rights Reserved.
31

Jess, Camilla, Lucy and Max are seated around a table at a restaurant. Two of them are facing
the entrance, and two are facing the back of the restaurant, where the bathroom is in view. I
know that Jess is sitting diagonally opposite Lucy, Max cannot see the bathroom and Camilla
is next to Jess.
Which of the following do I also know?
A. Lucy can see the bathroom
B. Camilla is sitting next to Max
C. Jess can see the entrance
D. Jess is sitting opposite Max

__________________________________________________________________________________
Copyright 2022 Into College Pty Ltd All Rights Reserved.
32

LOGIC TABLES
Logic tables are questions that involve the use of tables and grids to determine the answer. The
table is sometimes given; however students may also need to draw it out. This is a very common
thinking skills question and builds on previous topics such as interpreting information that is
heavily used.
Logic table questions usually present a list of information with a range of categories. Each
element in the category that exclusively matches with one another element in another category.
These elements must be matched up to determine the relationships between each element in
the categories.
Logic table questions require students to do step-by-step thinking to solve a problem, often
using process of elimination, working backwards and deductive reasoning. These questions
build a foundation for analysing data and information, using facts to solve problems and
drawing justified and reasonable conclusions. These problems not only equip the student’s
interpreting information skills, but also their logical and numerical reasoning skills as they
must make sense of clues and find logical orders to solve problems.

SKILLS & TIPS


Logic tables heavily rely on interpreting information, as well as logical reasoning skills.
To draw logic tables and solve these kinds of questions, students must first read the information
given to determine the number of categories and elements, i.e. Categories of ‘people’ and ‘age’,
and elements of ‘Amy, Jennifer, Lawrence’ and ‘12, 13, 14’ respectively.
Then, they are required to draw the table according to the number of categories and elements:
i.e., 3 people = 3 rows, and the related category of ‘age’ is represented in the 3 columns. Make
sure that each box is labelled.
Finally, the table can be filled in with a tick or a cross to rule out information. If something is
true for one element, i.e. Amy is 12, then the other categories can be ruled out, i.e. Amy cannot
be 13 or 14.

__________________________________________________________________________________
Copyright 2022 Into College Pty Ltd All Rights Reserved.
33

WORKED EXAMPLE
Three friends Jennifer, Claudia and Steph all like to eat different breakfast each. They also like
to get updates of the news via TV, radio or the newspaper and have their breakfast at 3 different
times: 6:30 am, 6:50 am and 7:15 am.
• The person who read the newspaper had breakfast later than the person who used the
radio
• Jennifer had her breakfast directly before the person who had toast
• Cereal was eaten 20 minutes after the person who watched the breakfast news on TV
• Claudia’s black coffee was not the breakfast taken last and prefers TV
What are Jennifer’s preferences for getting news updates and eating breakfast in the morning?
A. Radio, 6:30 am and toast
B. TV, 6:30 am, black coffee
C. Newspaper, 6:50 am and cereal
D. Radio, 6:50 am and cereal
Firstly, the logic grid can be drawn as shown with the appropriate categories and elements in
each category.

Next, the boxes in the grid can be ticked and crossed off according to the information given.
We can find information that directly tells us some existing or not existing relationships.
‘Jennifer had her breakfast directly before the person who had toast’ means that she cannot be
the latest time, nor does she eat toast.
‘Claudia’s black coffee was not the breakfast taken last and prefers TV’ directly tells us her
preferences, and that she is also not the latest time. These information can be filled in the grid
as shown:

__________________________________________________________________________________
Copyright 2022 Into College Pty Ltd All Rights Reserved.
34

With this information, you can cross out the rest of the column or row with a tick in it as it no
longer applies to anyone else. Once the grid is filled out according to the existing ticks, other
relationships can be deduced, such as Steph eating toast as that is the only option left. It can
also be noted that since cereal is eaten 20 minutes after the person who watches TV (who we
know is Claudia), Claudia’s breakfast time can only be 6.30am.

__________________________________________________________________________________
Copyright 2022 Into College Pty Ltd All Rights Reserved.
35

Once again, the grid can be filled out even more once we gain more information:

Since we know that the person who read the newspaper had breakfast later than the person who
uses the radio, Jennifer must use the radio and Steph must read the newspaper.

Finally, we can conclude that Jennifer uses the radio, and eats cereal at 6.50am for breakfast.

__________________________________________________________________________________
Copyright 2022 Into College Pty Ltd All Rights Reserved.
36

QUESTIONS
4 kids, Donald, Larry, Harold and Vincent all go to bed at different times (8PM, 8.30PM, 9PM
and 9.30PM) and are all different ages from 5 to 8. The oldest kid goes to bed 30 minutes earlier
than Harold. Donald sleeps later than Vincent, and he is 6 years old. The youngest boy sleeps
at 8PM while Donald sleeps at 8.30PM.
When does Larry go to bed?
A. 8PM
B. 8.30PM
C. 9PM
D. 9.30PM

Four women, Angelica, Cynthia, Mackenzie and Rachel went shopping, and each forgot one
item on their shopping list. The forgotten items are bread, coffee, detergent and milk, and the
amounts spent were $20, $30, $40 or $50. Angelica forgot to buy detergent. The woman who
spent $50 is either Rachel, or the person who forgot to buy milk. Mackenzie spent $30, and
Rachel spent $10 less than the person who forgot to buy coffee. Who spent $50 on their
shopping trip?
A. Angelica
B. Cynthia
C. Mackenzie
D. Rachel

My friends Rachel, Tina, Georgia and I went strawberry picking last weekend. We all used
different coloured baskets according to our favourite colour. Rachel used the white basket
and picked 4 more strawberries than the girl that used the red basket. The girl that picked 20
strawberries is either the girl who used the red basket or the girl who used the white basket.
Georgia carried 4 more strawberries than me, and I used the blue basket. After we counted
how many strawberries we picked, I know that each person either picked 20, 22, 24 or 26
strawberries.
What colour was Tina’s basket and how many strawberries did she pick?
A. Green, 20
B. Green, 26
C. Red, 20
D. Red, 26

__________________________________________________________________________________
Copyright 2022 Into College Pty Ltd All Rights Reserved.
37

ARGUMENTS

__________________________________________________________________________________
Copyright 2022 Into College Pty Ltd All Rights Reserved.
38

STRENGTHENING ARGUMENTS
Arguments is one of the topics under the Critical Thinking portion of Thinking Skills. It
includes 3 types of questions: strengthening arguments, weakening arguments, and mistakes,
flaws, and fallacies. Arguments, in the context of Thinking Skills, refer to statements that
provide a reason, or a set of reasons, given in support of an idea, action, or theory. To be able
to confidently and accurately answer these types of questions, students should be familiar with
interpreting information, finding the main conclusion of a given text, and pay attention to
keywords in a given text.
Strengthening argument questions are easy to identify in the Thinking Skills paper, as the
questions always ask the straightforward question: “Which of the following options, if true,
will strengthen the argument/claim the most?”
To identify the correct response, students must be able to identify the main conclusion of the
given text, identify keywords, and link the given information efficiently and reliably with the
new information that will be provided in the multiple-choice options. Students need to develop
a strong ability to interpret the text given to them in a short time frame. Interpreting information
is the process of understanding what information is relevant to the central argument and what
information is there as supporting evidence. Students are encouraged to note the frequently
mentioned ideas and words, which can help them identify the main contention.
This question type is the most common question type under the Arguments topic, and the skills
developed for this question type can be used in other Arguments questions. Therefore, it is
important to make sure students have a good foundation in solving strengthening arguments
questions.

SKILLS & TIPS


As mentioned above, students must be able to identify the main conclusion of the text and be
able to interpret new information in the multiple-choice options in relation to the text.
To identify the main conclusion, students should look out for repeated words that are used in
the text, and central ideas that the author of the text frequently mentions.
Summarising the text is a good first step to learn how to identify the main conclusion, as
students learn to eliminate information that is not essential to the main argument, but it is
important to note that summarising is not the same as identifying the main conclusion.
The multiple-choice options often include new information that is not mentioned in the text,
but will strengthen the argument if true. It is important for the students to understand that there
can be more than one option that strengthens the argument in the text, but the question will
always ask for the option that strengthens the argument the most. To identify the correct answer,
students should develop a habit of noting down the central idea, but also the supporting
evidence. If an option strengthens the argument, as well as the supporting evidence, it is more
often the right answer compared to another option that only strengthens the main conclusion.

__________________________________________________________________________________
Copyright 2022 Into College Pty Ltd All Rights Reserved.
39

WORKED EXAMPLE
There are some countries in the world where a high birth rate is a major worry, especially when
the country is less economically developed, and resources to feed and care for children are
scarce. But in other countries, the opposite is the case. The birth rate in several rich countries
is falling, at the same time as older people there are living longer. This will become a problem
as time goes on, because there will be far more old people who need looking after, but far fewer
younger people to do that. By allowing birth rates to fall so much, these countries are storing
up a problem for the future.
Which of these statements, if true, most strengthens the above argument?
A. Parents who have fewer children can devote more time and energy to the children they
do have.
B. Older people have much greater health needs, especially when they are over 85.
C. Older people do not always want to be dependent on their children and grandchildren.
D. Young people should see care of their parents and grandparents as their duty.

This text contains a lot of information. The first thing students should focus on identifying is
the main conclusion, and then the next step should be distinguishing the main conclusion from
the supporting evidence provided.
There are some countries in the world where a high birth rate is a major worry, especially
when the country is less economically developed, and resources to feed and care for children
are scarce. But in other countries, the opposite is the case. The birth rate in several rich countries
is falling, at the same time as older people there are living longer. This will become a problem
as time goes on, because there will be far more old people who need looking after, but far fewer
younger people to do that. By allowing birth rates to fall so much, these countries are storing
up a problem for the future.
The phrases that are bolded are the main ideas of the text. Both high birth rate and low birth
rate are problematic depending on the economic state of the country. This is the main
conclusion of the text. The underlined text are reasons provided in support of the main
conclusion. To strengthen the argument, the correct answer must a) strengthen the main
conclusion, and/or b) strengthen the supporting evidence. Now we turn to the given options.
Option A is an overall positive option. It does not support the main conclusion, nor does it
support the evidence. It points to the fact that having less children (i.e. low birth rate) is
advantageous, which is the opposite of the main conclusion.
Option B does not directly strengthen the main conclusion, it does however, strengthen the
supporting evidence. Since older people have greater needs, they will need young people to
look after them, and that will therefore highlight the problem of a low birth rate. This option is
not strong, but it does somewhat strengthen one side of the argument, so it is important to think
through the rest of the options.
Option C does not directly address the main conclusion, nor the supporting evidence. It does
contain some ideas that might fool students into thinking it’s relevant e.g. older people and
__________________________________________________________________________________
Copyright 2022 Into College Pty Ltd All Rights Reserved.
40

children, but knowing that older people don’t want to rely on the children or grandchildren is
irrelevant to the fact that low birth rate is problematic.
Option D is similar to Option C. It contains words that might confuse students, but at the end
the option is irrelevant to both the main conclusion and the supporting evidence. Even if young
people see caring for the elderly as their duty, low birth rate is still a problem and there will
still not be enough young people to take care of them.
The correct answer is B.

QUESTIONS
A scientist says: “Performing exercise each day can improve sleep quality.”
Which one of these statements, if true, best supports the scientist’s claim?
A. Those who exercise report feeling happier than those who don’t.
B. Many things other than exercise can also improve sleep quality.
C. Exercise allows the body to produce chemicals needed for the body to sleep.
D. Those who exercise report feeling more well-rested after sleep than others.

Nora says that if children are sitting in rows in a classroom, the teacher can have eye contact
with all of them while she is explaining something to them. When they look up, instead of
seeing the child opposite in a group and being tempted to talk, they see the teacher. So, sitting
in rows helps children to concentrate better on their work.
A. Which of these statements, if true, best supports the argument?
B. Educators have found that group work and projects increase student learning.
C. There is no way to maintain constant student and teacher eye contact.
D. Paying attention to what is being taught is key in being able to learn well.

Alan says that if children are sitting in rows in a classroom, the teacher can have eye contact
with all of them while she is explaining something to them. When they look up, instead of
seeing the child opposite in a group and being tempted to talk, they see the teacher. So, sitting
in rows helps children to concentrate better on their work.
Which one of the following, if true, would most strengthen Alan’s argument?
A. If desks are arranged in rows, children can all see visual aids more easily.
B. Sitting in groups of between four and seven makes discussion work easier.
C. Rows of desks take up no more classroom space than tables.
D. Some children are easily distracted whether they sit in rows or in groups.

__________________________________________________________________________________
Copyright 2022 Into College Pty Ltd All Rights Reserved.
41

WEAKENING ARGUMENTS
Weakening argument questions are very similar to strengthening arguments questions. The
questions involve the same elements: a given text with a central argument/claim, and the
prompt that asks students to identify the multiple-choice option that weakens the presented
argument/claim the most. Interpreting information is once again, an essential skill in weakening
arguments questions. Instead of focusing on what can strengthen the main conclusion or the
supporting evidence, now students must consider the option that weakens the main conclusion
and/or the supporting evidence the most.
The first steps involved in solving weakening arguments questions are identical to that of
solving strengthening arguments questions. Students must first distinguish between the main
contention of the text, and the supporting evidence that is given (interpreting information), and
then analyse the options that are provided accordingly. Though it may seem repetitive, the more
the students are able to practise identifying main conclusions and interpreting information, the
more confident they will be in Arguments questions. Strengthening and weakening arguments
are both very useful in daily life, and can encourage students to think critically about the
information that is presented to them.

SKILLS & TIPS


To learn about how to identify the main conclusion, refer to Strengthening Arguments. What
is unique to weakening arguments questions is that the information presented in the multiple
choice options can directly contradict the main conclusion and/or the supporting evidence,
whereas strengthening arguments questions are generally more subtle. This by no means makes
weakening arguments questions easier. The same principles also apply. There will often be
multiple options that weaken the argument/claim. It is important that the students understand
how to identify which option weakens the argument/claim the most. This will often mean that
the information provided in the multiple choice options weaken both the central argument and
the supporting evidence.

__________________________________________________________________________________
Copyright 2022 Into College Pty Ltd All Rights Reserved.
42

WORKED EXAMPLE
The health service urgently needs more blood donors to meet demands for blood transfusions.
Most people ignore this need yet are happy to receive the blood of other donors in emergencies.
To boost the blood bank (get more donors), transfusions should only be given to those who
have indicated they are donors or agree to become donors. In this system, receiving and giving
within the blood donation service would become clear in everyone’s mind and the blood bank
would be given all the donations it needs.
Which one of the following, if true, weakens the above argument?
A. Some people are ineligible to give blood due to many reasons.
B. Blood donations are at an all time low.
C. There is a high demand in certain blood types
D. Many people are wary of receiving blood because of possible infections
The first thing students should focus on identifying is the main conclusion, and then the next
step should be distinguishing the main conclusion from the supporting evidence provided. The
main conclusion of the text is stated in the middle of the text: “transfusions should only be
given to those who have indicated they are donors or agree to become donors.” The rest of the
information is there to give context to this conclusion.
Option A weakens the main conclusion as it weakens the assumption in the main conclusion.
The main conclusion assumes that everyone who needs transfusions would be able to donate
blood, whereas option A contradicts this assumption.
Option B is irrelevant to the main conclusion and merely restates the context given at the start
of the text.
Option C is irrelevant as blood types are not discussed in the text at all.
Option D is irrelevant to the main conclusion. The emphasis of the text is that there is a lack of
blood donors, stating that many people are wary of receiving blood can potentially weaken the
argument by diminishing the strength of the context, but it does not directly address the main
conclusion the way option A does.
The correct answer is A.

__________________________________________________________________________________
Copyright 2022 Into College Pty Ltd All Rights Reserved.
43

QUESTIONS
Despite the recent drop in numbers of people getting married, it is still recommended.
Married people are happier, wealthier and live longer. They have lower risk of disease and a
higher life expectancy. They smoke less, drink less and eat more healthily. Those who stay
married have more wealth than those who never marry, and surveys show that they are
happier and less likely to experience mid-life depression. Finding and committing to a
marriage partner is a goal well worth pursuing.
Which of the following, if true, most weakens the above reasoning?
A. Research suggests that being married boosts the chance of surviving cancer.
B. In many countries, being married gives advantages in terms of tax payments.
C. Surveys ask married couples with and without children.
D. Wealthier couples are more likely to marry than poorer couples.

Zoos are entirely unsuitable places for animals. People visit zoos to learn about animal
behaviour but the animals they see are likely to be behaving in abnormal and nervous ways
because of the cramped and unnatural conditions in which they are kept. Zoos should be
closed and the money saved should be used to protect the animals’ natural homes.
Which of the following, if true, would most weaken the above argument?
A. Humans living in cramped conditions can also become neurotic
B. Schoolchildren can learn a great deal about animals from visiting zoos
C. The protection of wildlife habitats is very costly
D. Zoos enable endangered species to survive by breeding them in captivity and then re-
introducing them to the wild

A health specialist has suggested that dance lessons should not be an important part of school
life, and that school children should spend more time playing sport and not dancing.
Which one of these statements, if true, best weakens the health specialist’s claim?
A. Dancing has not been taught in schools in the past.
B. Some children cannot afford equipment to participate in sports.
C. Dancing improves children’s metal abilities and class relationships.
D. Not everyone enjoys dancing and prefers other activities.

__________________________________________________________________________________
Copyright 2022 Into College Pty Ltd All Rights Reserved.
44

MISTAKES, FLAWS, AND FALLACIES


Mistakes, flaws, and fallacies require students to recognise a bad argument. A good argument
has two key characteristics: true, and valid. A good argument provides factual information, and
the conclusions of the argument logically follows the premises. When either characteristic is
compromised, the argument is no longer considered a good argument and it is considered
flawed or unsound.
These types of questions are often tricky for students as they often commit these fallacies
themselves in their daily lives. Even if the students are able to recognise that there is something
not quite right about the argument, they often have trouble articulating what exactly is unsound
about the argument. Nevertheless, mistakes, flaws, and fallacies questions have a lot less
variety compared to strengthening and weakening arguments questions, and the more the
students practice, the easier the questions will be. This question type falls under the Arguments
topic, but is relatively distinct to strengthening and weakening arguments questions.
It is therefore important for students to develop the skill set for answering mistakes, flaws, and
fallacies questions. The skills gained in this topic can also help students critically think about
their own arguments, and the arguments of other people, so that students grow to value a
logically consistent argument rather than falling into the trap of logical fallacies.

SKILLS & TIPS


Mistakes, flaws, and fallacies questions usually contain one of these two mistakes: incorrect
deduction and illogically high certainty. In other words, the arguments students are presented
with in the text can have a potentially true conclusion, but have neglect to consider other
possibilities, or the conclusions can have a higher certainty than what is considered probable.

WORKED EXAMPLE
Mr Greene: “I know that eight children in my class are vegetarian, but ten children ordered the
sandwich that does not contain meat for lunch today. So, there must be more than eight
vegetarians in my class.”
Which of the following shows the mistake Mr Greene has made?
A. He did not count the children in other classes who needed to order sandwiches.
B. He did not know which eight children in particular were vegetarian.
C. Some children who used to be vegetarian are no longer vegetarian.
D. Some children who are not vegetarian happen to want vegetarian sandwiches.
This is an example where there is incorrect deduction. Below is the analysis of all the options.
Option A is irrelevant as Mr Greene is talking about students in his class, not other classes.

__________________________________________________________________________________
Copyright 2022 Into College Pty Ltd All Rights Reserved.
45

Option B is irrelevant because even if Mr Greene does know which students are vegetarian, he
still made a mistake in his reasoning, and there are still 10 vegetarian orders instead of 8.
Option C can be true, but the problem Mr Greene is facing is that there are more than 8
vegetarian sandwiches ordered, not less.
Option D is the correct response as Mr Greene has neglected to account for students who just
prefer the vegetarian option instead of the meat option, even if they are not vegetarian.

QUESTIONS
Schools are encouraging students to learn poetry by heart in order to recite it for a new
competition. Schools hope to boost the confidence of young people and increase their
awareness of poetry. By reciting poetry, young people will improve their appreciation of
poetry.
Which one of the following identifies a flaw in the above argument?
A. You don’t have to recite poetry to appreciate it.
B. Students are already aware of poetry.
C. Awareness is not the same as appreciation.
D. The young people who are willing to recite poetry are already confident.

"There are more police officers than ever before in the history of this country. We have more
police officers and more community support officers. That is why, last week, crime was
down. Crime is down 30 percent."
Which of the following states the flaw in the argument above?
A. Community support officers don't have the same powers as police officers.
B. Crime levels can be measured in different ways.
C. Crime could have fallen if there had been no increase in policing.
D. The fact that crime was down last week does not mean it will stay down.

Jose has a thesaurus that can show the synonyms of any English word.
Jose: “If you tell me a word, you don’t need to tell me its definition – I can search it up in the
thesaurus and find the meaning from the synonyms.”
Which one of the following sentences shows the mistake Jose has made?
A. There are some words that can be altered by prefixes or suffixes.
B. There are some words that are archaic and no longer used.
C. There are some words that have multiple meanings.
D. There are some words that do not have synonyms.

__________________________________________________________________________________
Copyright 2022 Into College Pty Ltd All Rights Reserved.
46

DRAWING CONCLUSIONS

__________________________________________________________________________________
Copyright 2022 Into College Pty Ltd All Rights Reserved.
47

Drawing conclusions questions require students to interpret the information provided and make
a judgement about the reasoning used in the question. Typically, these questions involve a
background passage and two conclusions, asking students to decide whose reasoning is correct.
These questions involve interpreting the given information then using deduction and induction
to judge the validity of the conclusions provided. Drawing conclusions is highly relevant and
applicable to other subjects and everyday life. These questions test the student’s ability to read
information quickly and gather the important information.
Drawing conclusion questions are easy to identify as they usually follow this structure:
Background info: …
Person A “…”
Person B: “…”
If the information given is true, then whose reasoning is correct?
A. Person A only
B. Person B only
C. Both person A and person B
D. Neither person A nor person B

SKILLS & TIPS


When Interpreting information:
• Look for keywords and highlight them
o Some keywords include: but, however, some, most, all, never etc.
• Locate topic sentence
o Usually this is the first sentence of an argument
o This is the sentence that outlines the main argument of the paragraph
• Filter out filler sentences
o These are sentences that do not offer key points or any new ideas
It is important to make sure that we are 100% certain that our conclusion stems from the
information given in the question. This means that the conclusion cannot stem from external
information or any other knowledge that you may know.

__________________________________________________________________________________
Copyright 2022 Into College Pty Ltd All Rights Reserved.
48

WORKED EXAMPLE
Exercise is able to improve mood and sleep quality, but is not the only factor in someone’s
overall health.
Ivan: “I have exercised every day for the past week, which is why I have been in a great mood
and have had good sleep. Exercise must be the reason for this!”
Hector: “I have not exercised at all, but have still been in a great mood and have had restful
sleep. Exercise has no benefits!”
If the first statement is true, whose reasoning is correct?
A. Ivan only
B. Hector only
C. Both Ivan and Hector
D. Neither Ivan nor Hector
Option D is the right answer, as both exercise and other factors contribute to mood and sleep
quality. This means that even though it could have played a part in Ivan’s health, it may not be
the only reason. Similarly, no exercise could have increased Hector’s health, but there may be
other factors at play.

QUESTIONS
Cyanobacteria are the only bacteria that can photosynthesize.
Jack: “If you see some bacteria and they cannot photosynthesize, they must not be
cyanobacteria.”
Jackie: “If you see a living thing that is photosynthesizing, and it is not bacteria, then it is not
cyanobacteria.”
If the first statement is true, whose reasoning is correct?
A. Jack only
B. Jackie only
C. Both Jack and Jackie
D. Neither Jack nor Jackie

__________________________________________________________________________________
Copyright 2022 Into College Pty Ltd All Rights Reserved.
49

To become a good doctor, you need to be intelligent and also good at communicating your
ideas.
Andrew: “I have a high IQ and I am great at expressing myself, so I can become a good
doctor.”
James: “I always get straight As at school so I can become a good doctor.”
If the information above is correct, whose reasoning is correct?
A. Andrew only
B. James only
C. Both Andrew and James
D. Neither Andrew nor James

Annabelle is the only female student in the class with a brother.


Student A: “My parents have three children that are all daughters. My name cannot be
Annabelle.”
Student B: “My parents have a son and two daughters. I have only one sister. My name must
be Annabelle.”
If the information above is true, whose reasoning is true?
A. Student A
B. Student B
C. Both Student A and B
D. Neither Student A nor B

__________________________________________________________________________________
Copyright 2022 Into College Pty Ltd All Rights Reserved.
50

QUANTITATIVE REASONING

__________________________________________________________________________________
Copyright 2022 Into College Pty Ltd All Rights Reserved.
51

Quantitative reasoning is where basic maths skills are applied in order to analyse and interpret
real-world issues or scenarios. This means we are combining mathematical reasoning with
problem solving in order to arrive at an answer and draw conclusions relevant to the scenario
given. Although this may seem similar to mathematics, it is so much more than just that. It is
a type of logical reasoning topic that requires mathematical skills to solve!
The questions under quantitative reasoning can be separated into 2 branches:
• Numerical working
• Graphs and diagrams

NUMERICAL WORKING
Numerical working is related to numeracy and the ability to apply mathematical concepts in
any kind of scenario. It involves the construction of mathematical equations based on the
understanding of numbers, counting, solving problems and through the analysis of the
questions’ information. Specifically, it is used to assess the student’s ability to use these skills
to generate equations based on their interpretation of the questions and hence solving them.
This goes hand in hand with interpreting information as the student must make sense of the
information first before constructing an equation. This requires the students to note and analyse
the information and/ or tables provided to correctly answer the question. However, despite the
difficulties of these questions, the mathematical skills required are not of high level which is
why understanding the question is the key to arriving at the correct answer.

SKILLS & TIPS


• Understand what the question is asking for
o Are there conditions?
o Are there premises?
o What are we looking for?
• Choose the appropriate method to solve
o What kind of equation do we want to write?
o Do we need diagrams?
o Do we need graphs?
• Look for keywords and subject
o Some key words include: but, however, some, most, all, never etc.
o Determine conditions
• Take note of numbers, units etc.
o Make sure all units are the same!

__________________________________________________________________________________
Copyright 2022 Into College Pty Ltd All Rights Reserved.
52

Remember:
• Steps to solving quantitative reasoning questions differ according to the question!
o You must be flexible and change your method according to the type of question!
• Generally:
o Interpret the question first
o Write out equations according to the information
o Eliminate possible options
Note: you can always use trial and error although it is time consuming!

WORKED EXAMPLE
Linda has been getting some advice from her bank on how to invest her money to be able to
make the most of it. She was told that she needs to calculate how much money she has left
from her salary each month once she has paid all her bills. Then she should invest up to 70%
of that amount.
Linda earns $2200 each month and must pay $800 in rent. She approximated her other expenses
and bills to be $650 per month.
What is a possible amount that Linda is advised to invest in each month?
A. $750
B. $500
C. $620
D. $550
Work out how much she has remaining first: 2200 – 800 – 650 = $750
Next, it says she should invest UP TO* 70%.
*UP TO means anywhere from 0-70% of her remaining amount. It can be below 70% but
cannot exceed 70%.
70% = 7/10
750 ÷ 10 = 75 = 1/10
7/10 = 75 x 7 = 525
Since the question stated UP TO $525, the only amount under $525 is B.
The correct answer is B.

__________________________________________________________________________________
Copyright 2022 Into College Pty Ltd All Rights Reserved.
53

QUESTIONS
At a reptile park, there are spiders, lizards, tropical birds and snakes. It is known that spiders
have 8 legs, lizards have 4 legs, tropical birds have 2 legs and snakes have no legs. There are
15 of these animals altogether, with 38 legs in total.
What is the highest number of snakes that can be at the reptile park?
A. 8
B. 9
C. 10
D. 11

Three friends, Jane, Katherine and Lily, all gave birth on the same day.
Jane's daughter was the first to arrive, at 10:10, followed by Katherine's son 105 minutes later.
Finally, Lily had her son 112 minutes before midnight.
What was the time interval between the births of the two boys?
A. 8 hours 58 minutes
B. 9 hours 15 minutes
C. 10 hours 13 minutes
D. 10 hours 27 minutes

Olivia is making cards for all of her friends. She needs to buy blank cards to draw her designs
on. The prices for different sizes of cards are shown below:
Price
Pack of 25 Pack of 50 Pack of 75 Pack of 100
Small $15 $25 $42 $75
Medium $17 $30 $44 $80
Large $20 $35 $50 $90

Olivia wants to draw her design on 47 small cards and 80 medium cards.
What is the lowest amount that Olivia can pay for the cards?
A. $78
B. $85
C. $86
D. $105

__________________________________________________________________________________
Copyright 2022 Into College Pty Ltd All Rights Reserved.
54

GRAPHS & DIAGRAMS


Another very prominent subtopic of quantitative reasoning is graphs and diagrams. Graphs are
drawings that represent mathematical information using a variety of methods such as lines,
shapes and colours. In thinking skills, the 3 main kinds of graphs a student may encounter are:
line graphs, bar graphs and circle graphs/ pie charts.
Learning to interpret, understand and construct graphs is not only important in solving thinking
skills questions but will also help students to build a strong foundation for future studies.
Diagrams are also used in quantitative reasoning as a visual representation of the information
given in the question or acts as supporting information that must be interpreted in order to solve
the question. Depending on the question, the diagrams may be given, or the student must draw
a diagram from scratch to aid in solving the quantitative reasoning question. The main types of
diagrams in quantitative reasoning questions include Venn diagrams, informative diagrams,
timetables and visual aids
Learning to interpret and understand diagrams along with verbal information will further aid
the student in their problem-solving skills and allow students to learn to combine multiple skills
to solve questions.

SKILLS & TIPS


Graphical questions:
• Understand the individual components of graphs
o Interpret the components together to answer the question
• Take note of the axis and units
o Make sure axes are correct - horizontal and vertical
o Identify each section of the graph correctly
• See if there are relationships
o Look out positive? Negative?
• Take note of keywords
o Relate keywords to the graphs
Venn Diagrams:
1. Find the total number of items
2. Draw a Venn diagram and label it
3. Start filling in numbers from the centre
4. Fill in the single category numbers, subtracting the overlaps
5. Fill in the no category numbers
6. Answer the question (ATQ)
Timetables:
• To read, look at the times on each line
• Some stops could be blank!
__________________________________________________________________________________
Copyright 2022 Into College Pty Ltd All Rights Reserved.
55

o It is skipped!
• When solving, take note of conditions for the timetables
o Based on the condition, can anything be eliminated?
o Any tricks to make things easier?
Informative diagrams:
• Relate the diagram to the question
• Interpret the visuals and information given in the diagram

WORKED EXAMPLE
Jennifer saved up for 8 months to buy a $1000 gold chain. The table shows how much she
saved in each of the 7 months leading up to December. By the end of December, she had exactly
enough money saved to buy the chain.
May June July August September November December
$150 $125 $175 $150 $125 $50 $150

Which of these bar charts, with the axes suitably labelled, could represent how much Jennifer
saved each month?

Since the question is asking for a graph with 8 months and there are only 7 shown in the table,
step 1 would be to find out how much Jennifer saved in December:
• 1000 – (150 + 125 + 175 + 150 + 125 + 50 + 150) = $75
Find the relationships between the numbers:
• May (1) = Aug (4) = Nov (7)
• June (2) = Sep (5)
• Oct (6) = lowest
Only B is possible since it is the only graph with bars 1 = 4 = 7, bars 2 = 5 and bar 6 as the
shortest.

__________________________________________________________________________________
Copyright 2022 Into College Pty Ltd All Rights Reserved.
56

QUESTIONS
Jean lives in Annfield and wants to go by bus to Elton to arrive before 16:00 to meet with her
friends and enjoy a night out. On the say, she wants to stop in Chester for 40 minutes to do
some shopping as it was one of her friend’s birthday.
Below is the bus timetable:
Bus Timetable
ANNFIELD 13:08 13:17 14:10 14:32 14:42
BOWES 13:40 13:50 14:30 15:02 15:18
CHESTER 14:13 14:28 14:58 15:23 15:50
DURHAM 14:28 14:50 15:16 15:38 16:06
ELTON 14:50 15:12 15:38 15:56 16:20

What is the latest bus she can catch to ensure she can do some shopping and also meet her
friends on time?
A. 1:08 pm
B. 1:17 pm
C. 2:10 pm
D. 2:32 pm

Oliver is on an interstate bus. After a short nap, he woke up just in time to see the sign on the
side of the motorway:
Ascot 12 km
Canning Vale 27 km
Subiaco 33 km
Beldon 49 km
Melville 54 km

The last thing he remembers seeing before he fell asleep is this sign:
Kenwick 8 km
Eden Hill 12 km
Clarkson 24 km
Ascot 58 km
Canning Vale 73 km

How far has the bus travelled while Oliver was asleep?
A. 46 km
B. 54 km
C. 58 km
D. 73 km

__________________________________________________________________________________
Copyright 2022 Into College Pty Ltd All Rights Reserved.
57

My company has 1600 employees. Of those employees, 900 have studied science. ⅕ of those
who studied computer science also studied english. That group of computer science and
maths student employees makes up half of the total employees who studied mathematics.
How many employees have studied neither computer science nor mathematics?
A. 160
B. 200
C. 420
D. 520

A health specialist has suggested that dance lessons should not be an important part of school
life, and that school children should spend more time playing sport and not dancing.
Which one of these statements, if true, best weakens the health specialist’s claim?
A. Dancing has not been taught in schools in the past.
B. Some children cannot afford equipment to participate in sports.
C. Dancing improves children’s metal abilities and class relationships.
D. Not everyone enjoys dancing and prefers other activities.

__________________________________________________________________________________
Copyright 2022 Into College Pty Ltd All Rights Reserved.
58

ABSTRACT REASONING

__________________________________________________________________________________
Copyright 2022 Into College Pty Ltd All Rights Reserved.
59

2D SPATIAL AWARENESS
A main component of the Thinking Skills abstract reasoning topic in spatial awareness. Spatial
awareness questions are divided into two subcategories – 2D spatial awareness and 3D spatial
awareness.
2D spatial awareness is defined as the ability to understand, reason and remember spatial
relations among 2D objects or space. These questions assess a student’s capacity to manipulate
2D objects, spot patterns between shapes, and to visualise movements and change in those
shapes. This is a crucial and relevant skill as spatial awareness is something we exhibit
everyday as we interpret objects and shapes in our daily lives. These skills learnt for spatial
awareness questions are valuable in many real-world situations and can be improved with
practice.

SKILLS & TIPS


2D spatial awareness questions require four main skills:
1. Spatial perception - the ability to see and understand shapes and relationships between
them.
o E.g. seeing that something is further away or closer to you
2. Spatial visualisation - being able to visualise complicated multi-step processes
involving shapes and objects.
o E.g. folding a net into a cube
3. Mental folding - visualising what an object looks like after folding
o E.g. paper cut outs
4. Mental rotation - visualising the rotating of an object in space.
o E.g. spotting the odd shape out from rotating it

Another skill required for this topic is deduction. Specifically, we can use deduction to deduce
the relationships between shapes, and break the question down into smaller steps, using some
of the following methods:
1. Sides/edges
o E.g. matching the sides of different shapes
2. Sections on a board
o E.g. finding the missing section on a board with a repeated pattern
3. Segments of digital displays
o E.g. broken segments of digital clock displays
4. Smaller pieces creating a larger shape
o E.g. forming new shapes from several smaller shapes

__________________________________________________________________________________
Copyright 2022 Into College Pty Ltd All Rights Reserved.
60

WORKED EXAMPLE
In the committee room of my cricket club there are four tables, all identical to the
one shown below.

Which of the following arrangements is it not possible to make by putting four of these tables
together?

Break each of the four larger shapes down according to the given shape. This will reveal that
shape D has a missing square in the centre and therefore is not possible.

__________________________________________________________________________________
Copyright 2022 Into College Pty Ltd All Rights Reserved.
61

QUESTIONS
When a mixed doubles tournament is held at my tennis club, partners are chosen by the
following method.
The appropriate number of tokens, all with a different shape, are placed into a box, and the
male participants all pick one out. Each token has an identical 'twin' in a second box, from
which the female participants all select one. Each pair of identical tokens fit together to make
a complete disc. Couples with identical tokens partner each other in the tournament.
Which one of the following cannot be a token used by my tennis club?

__________________________________________________________________________________
Copyright 2022 Into College Pty Ltd All Rights Reserved.
62

The floor of the reception area of the head office of Stott and Walsh is tiled as shown below:

Which one of these tiles could not be used to replace a damaged tile on this floor?

Instructions were given to shade 5 particular squares on a grid of 16 squares. One student
shaded in the squares as instructed, but the other four got exactly 1 square wrong.
Which one of the five grids below shows the correct shading?

__________________________________________________________________________________
Copyright 2022 Into College Pty Ltd All Rights Reserved.
63

3D SPATIAL AWARENESS
3D spatial awareness is the other area under spatial awareness questions and builds on concepts
and skills similar to those used for 2D spatial awareness questions.
3D spatial awareness is defined as the ability to understand, reason and remember spatial
relations among 3D objects or space. These questions assess a student’s ability to manipulate
3D objects and to visualise movements and change in those shapes. This is a crucial and
relevant skill as spatial awareness is something we exhibit everyday as we interpret objects and
shapes in our daily lives. These skills learnt for spatial awareness questions are valuable in
many real-world situations and can be improved with practice.
There are two types of 3D spatial awareness questions:
1. Folding
o Being able to fold nets into 3D
o Being able to keep track of the faces of the net in 3D
o Skills: spatial perception, spatial visualisation, mental folding

2. 3D perspective
o Viewing 3D structures from different angles
o Being able to deduce how a 3D object looks from another perspective or angle
o Skills: spatial perception, spatial visualisation, mental rotation

SKILLS & TIPS


3D spatial awareness questions require four main skills:
1. Spatial perception - being able to visualise and understand 3D shapes and relationships
between them.
o E.g. seeing that something is further away or closer to you
2. Spatial visualisation - visualise complicated multi-step processes involving 3D shapes.
o E.g. folding and rotating a 3D object
3. Mental folding - visualising what an object looks like after folding.
o E.g. folding nets into 3D shapes
4. Mental rotation - rotating a 3D object in space.
o E.g. seeing new perspectives of a rotated object
Note: deduction is also required for 3D spatial awareness questions.

__________________________________________________________________________________
Copyright 2022 Into College Pty Ltd All Rights Reserved.
64

WORKED EXAMPLE
This is the view from above of the large three-dimensional logo situated in the forecourt of the
head office of Justin Thyme Enterprises:

It was sculpted from a single cube of marble and has a uniform cross-section from top to bottom.
Which of the following is not a possible side view of the logo?

The possible side views of the logo means viewing it from all directions, from the top view,
bottom view and from either side of the “J” or the “E”.
From the top view, there will be a long extended flat surface with two bars of the curve of the
“J”. This matches Option D.

__________________________________________________________________________________
Copyright 2022 Into College Pty Ltd All Rights Reserved.
65

From the bottom view, there will be a longer segment with three shorter segments sandwiched
by another longer segment. This matches Option C.

From the “J” side view, there will be a longer segment with one shorter segment.

From the “E” side view, there will be three shorter segments that sandwich two longer segments.
This matches Option A.

Thus, the only option left out is Option B.

__________________________________________________________________________________
Copyright 2022 Into College Pty Ltd All Rights Reserved.
66

QUESTIONS
A view of a single level of a building is shown below:

Which one of the following drawings represents the floor plan for this level of the building?

In maths class, Nicky was learning about 3D objects such as shapes. To help them understand
the concept better, they were asked to draw nets and cut them out to create cubes. Each
student added different designs to their nets and Nicky added black triangles to her design.
This is the final cube that Nicky created from her net.

Which of the nets can be folded to create Nicky’s cube?

__________________________________________________________________________________
Copyright 2022 Into College Pty Ltd All Rights Reserved.
67

A number of identical boxes are piled up, waiting to be loaded into a delivery van. Amy and
Ben are about to load them into the van.
The appearance of the pile of boxes and the positions of Amy and Ben at present are shown
below:

Amy's view of the pile is:

Which one of the following could not be Ben's view of the pile?

__________________________________________________________________________________
Copyright 2022 Into College Pty Ltd All Rights Reserved.
68

SEQUENCE RECOGNITION
Abstract reasoning is non-verbal and visual spatial reasoning ability. It involves identifying
patterns, spotting trends, and ignoring distracting information to arrive at the right answer.
Abstract reasoning is an essential part of developing critical thinking and problem-solving
skills. These skills are not only important for Thinking Skills but are transferable across other
subjects such as Maths and English.
There are two main branches for abstract reasoning questions:
• Sequence recognition
• Spatial awareness
In Thinking Skills, sequence recognition questions is an abstract reasoning topic that focuses
on identifying and interpreting figure sequences. There are 2 main types of sequence
recognition questions:
1. Odd one out
o The question provides four sequences and students need to identify the one that
does not belong.
2. Missing shape
o The question provides a series of shapes and students need to find the shape that
comes next in the sequence.
Other sequence questions can also include combining different shapes.
Unlike General Ability, sequence recognition questions in Thinking Skills have lots of
background information and a scenario. The questions typically involve rotation, symmetry,
shape/patterning, colour and reflections.

SKILLS & TIPS


Sequence recognition questions in Thinking Skills require multiple skills, including:
Sequence recognition
Interpreting information

__________________________________________________________________________________
Copyright 2022 Into College Pty Ltd All Rights Reserved.
69

There are 4 main steps to solving sequence recognition questions:


1. Interpret the question
o The question will have very important information! The information given in
the question will give us an idea of the premises we must follow.
2. Determine sequence type
o What kind of sequence Q is it? Odd one out or missing shape?
3. Determine the relationship(s)
o Find the relationships between the shapes by looking at:
§ Rotation
§ Colour
§ Shape/patterning
§ Symmetry
§ Reflections
4. Process of Elimination
o Depending on sequence type, eliminate options to find correct answer

WORKED EXAMPLE
The other day I saw an alien spacecraft, which had some strange markings on the underside,
hovering overhead. My three friends and I each made a drawing of these markings as the craft
rotated slowly above us.
Three of us drew them correctly. Which of the following drawings is wrong?

Option C is the right answer.


• Type of question: odd one out
• Type of relationship: rotation
Shape A rotated 90 degrees anti-clockwise produces B. B rotated 90 degrees anti-clockwise
produces D. However, to get C, we need to flip B horizontally.
Thus, C is the odd one out as it cannot be produced via rotation.

__________________________________________________________________________________
Copyright 2022 Into College Pty Ltd All Rights Reserved.
70

QUESTIONS
A window in my new house is made of six panes of glass. Half of each pane of glass is coloured
and half is clear. Unfortunately, one of the panes is missing so the window currently looks as
shown below.

I want to fill in the missing pane of glass so that at every edge the colours match. Which one
of the panes of glass could I use to fill the gap (the pane can be rotated or turned over)?

Thomas is designing a puzzle made of four pieces. Each piece is made of three squares and
when they are put together correctly they will form a 3 × 4 grid of squares. The design will
also have two lines of symmetry. Three of the pieces are shown below.

Which of the following could be the final piece?

__________________________________________________________________________________
Copyright 2022 Into College Pty Ltd All Rights Reserved.
71

Kaye is making a new logo for her company. She makes the shape below by gluing together 8
squares and then she cuts along the dashed line.

She separates the two pieces, paints them grey on both sides and waits for them to dry. Then
she rearranges them, without overlapping.
Which of these designs can she not make?

__________________________________________________________________________________
Copyright 2022 Into College Pty Ltd All Rights Reserved.
72

PRACTICE QUESTION ANSWERS


LOGICAL REASONING
Interpreting Information
1. B
2. B
3. A
Euler/Syllogisms
1. D
2. C
3. C
Logic Lists
1. D
2. B
3. D
Logic Tables
1. C
2. B
3. C

ARGUMENTS
Strengthening Arguments
1. D
2. C
3. A
Weakening Arguments
1. D
2. D
__________________________________________________________________________________
Copyright 2022 Into College Pty Ltd All Rights Reserved.
73

3. C
Mistakes, Flaws, Fallacies
1. A
2. C
3. D

DRAWING CONCLUSIONS
1. C
2. A
3. C

QUANTITATIVE REASONING
Numerical Working
1. B
2. C
3. B
Graphs and Diagrams
1. B
2. A
3. D

ABSTRACT REASONING
2D Spatial Awareness
1. C
2. B
3. D
3D Spatial Awareness
1. D
2. A
3. D
Sequence Recognition
1. C
2. C
3. D

__________________________________________________________________________________
Copyright 2022 Into College Pty Ltd All Rights Reserved.
74

IntoTS & Into College

__________________________________________________________________________________
Copyright 2022 Into College Pty Ltd All Rights Reserved.
75

IntoTS
Thinking Skills is a collection of questions involving real-world issues that challenge students’
abilities in critical thinking and solving novel problems.
Our course fosters students’ abilities in both these targeted and overarching skills, preparing
them not only for logical thinking and problem-solving, but also in fundamental skills for
analytical thinking and decision making. Further, these foundational abilities will benefit
students throughout high school and beyond, even appearing in subsequent exams such as the
UCAT.

LEARNING FRAMEWORK
Our course spans across four terms, with ten lessons per term. Throughout the course, each
term targets a range of topics related to Thinking Skills, including new content such as an
introduction to formal logic as well as specific methods for interpreting and solving Thinking
Skills questions. Each term, our content builds upon similar concepts from prior lessons to
allow students to form connections with past material and gain an understanding of the
materials’ complexity.
We believe in allowing students’ own curiosity and interests to guide their learning, with group
class activities as well as independent investigation to create a challenging, yet enjoyable
learning environment. This also helps students in translating difficult logic concepts into
everyday examples - a key emphasis in Thinking Skills.

DEVELOPING STUDENTS’ ABILITIES


Into Thinking Skills not only provides students with greater exposure to challenging questions,
but also introduces ideas of formal logic and reasoning that inform the questions themselves.
We believe in fostering students’ own curiosity and love of learning and ensure that students
understand the underlying concepts these questions are based upon. Each lesson targets a
specific topic related to Thinking Skills, whether it be an introduction to types of reasoning or
how to use Venn and Euler diagrams. Not only this, we also develop students’ study skills and
problem-solving tools, equipping them to critically analyse novel problems and scenarios.

__________________________________________________________________________________
Copyright 2022 Into College Pty Ltd All Rights Reserved.
76

HOMEWORK & EXAM PREPARATION


Students in our course are assigned homework each week as revision from the lesson. Our
homework contains three main sections:
1. Note-taking
Due to the new style of Thinking Skills questions, students are introduced to a variety of new
concepts such as formal logic and types of reasoning. These notes allow students to better
absorb novel content, while developing their note-taking skills and allowing them to take
responsibility for their own learning.
2. Questions
The homework questions assess similar problem-solving skills and critical thinking
corresponding to each lesson. However, these questions present novel scenarios and draw on
students’ abilities to think flexibly, much like the true Thinking Skills exam.
3. Study Skills
To foster students’ broader abilities in critical thinking and development, Study Skills
challenges students in a wide range of activities, from mind-mapping to using de Bono’s six
thinking hats.

Each term, there are also two in-class examinations to assess students’ understanding and
progress of both the material and problem-solving abilities. These tests correspond to the
content taught in the term and are a valuable source in helping each student target their personal
strengths and weaknesses.

__________________________________________________________________________________
Copyright 2022 Into College Pty Ltd All Rights Reserved.
77

__________________________________________________________________________________
Copyright 2022 Into College Pty Ltd All Rights Reserved.

You might also like